LSAT and Law School Admissions Forum

Get expert LSAT preparation and law school admissions advice from PowerScore Test Preparation.

 Administrator
PowerScore Staff
  • PowerScore Staff
  • Posts: 8917
  • Joined: Feb 02, 2011
|
#40795
Complete Question Explanation
(The complete setup for this game can be found here: lsat/viewtopic.php?t=4676)

The correct answer choice is (A)

The question stem creates an LT block, and such a block can occur only under Template #2. In that template, L dives first, T dives second, and O dives last. The remaining three divers—W, P, and Z—can dive in any order in the third, fourth, and fifth positions:

PT63_Game_#2_#10_diagram 1.png
As shown in the diagram, O must dive last, and thus answer choice (A) cannot be true and is thus correct.

Note that, not surprisingly, each of the four incorrect answers addresses W, P, or Z, the three variables with uncertainty in this situation.
You do not have the required permissions to view the files attached to this post.

Get the most out of your LSAT Prep Plus subscription.

Analyze and track your performance with our Testing and Analytics Package.